You are on page 1of 36

226

New Pattern Q/s

Q1. The following bar graph represents the percentage increase in admission of
students in colleges A, B, C and D in the following years as compared to the
previous year. If the number of students admitted in each of college A and C in
2020 is 'x' and the number of students admitted in each of college B and D in 2022
is '2x', then the number of students admitted in college A and B together in 2021
is approximately what percent of the number of students admitted in college C
and D together in 2021?
निम्ननिखित बार ग्राफ निछिे वर्ष की तुििा में अगिे वर्ों में कॉिेज ए, बी, सी और डी में
छात्रों के प्रवेश में प्रनतशत वृखि कर दशाषता है । यनद 2020 में प्रत्येक कॉिेज A और C में
प्रवेनशत छात्रों की सोंख्या 'x' है और 2022 में प्रत्येक कॉिेज B और D में प्रवेनशत छात्रों की
सोंख्या '2x' है , तर कॉिेज A और B में प्रवेनशत छात्रों की सोंख्या 2021 में कुि नमिाकर
कॉिेज C और D में प्रवेश िािे वािे छात्रों की सोंख्या, 2021 में कुि नमिाकर िगभग
नकतिा प्रनतशत है ?
A. 77%
B. 80%
C. 85%
D. 89%
E. None of these

Q2. In column I, two questions are given with variables X and Y and in column II,
some set of values, quadratic equations, or two linear equations are given. You
have to find that the values from sets or roots of the quadratic equation or linear
equation from each row of column II is/are satisfies the value of X and Y of each
question of column I :

Note - The value of X is always greater than the value of Y.

कॉिम I में, चर X और Y के साथ दर प्रश्न नदए गए हैं और कॉिम II में, मािरों का कुछ सेट,
निघात समीकरण या दर रै खिक समीकरण नदए गए हैं । आिकर यह िता िगािा हरगा नक
कॉिम II की प्रत्येक िोंखि से निघात समीकरण या रै खिक समीकरण के सेट या जडरों के
माि कॉिम I के प्रत्येक प्रश्न के X और Y के माि कर सोंतुष्ट करते हैं : िरट - X का माि सदै व
Y के माि से अनिक हरता है ।
A. A-I & B- II, III, IV
B. B-III & A- I, II, IV
C. A-I, II & B- III, IV
D. A-I, IV & B- III, II
E. A-I, III & B- II, IV

Q3. Area of a rectangle P is (___ + 29/119)% less than the area of rectangle Q. The
perimeter of rectangle Q is 14 cm more than the perimeter of a square whose
area is ___ cm2 less than the area of rectangle P. Length and breadth of rectangle
P are in the ratio 9 : 8 respectively and side of square is equal to the breadth of
rectangle P. Area of square is 576 cm2 and the respective ratio of breadth of
rectangle Q to the side of square is 7 : 8. The difference between sides of
rectangle Q is ___ cm.
Which of the following option is possible to fill the given blanks in same order?
आयत P का क्षेत्फि आयत Q के क्षेत्फि से (___ + 29/119)% कम है । आयत Q का
िररमाि उस वगष के िररमाि से 14 सेमी अनिक है नजसका क्षेत्फि आयत P के क्षेत्फि से
___ सेमी2 कम है । आयत P की िोंबाई और चौडाई का अिुिात क्रमशः 9:8 है और वगष की
भुजा, आयत P की चौडाई के बराबर है । वगष का क्षेत्फि 576 सेमी2 है और आयत Q की
चौडाई और वगष की भुजा का सोंबोंनित अिुिात 7 है : 8. आयत Q की भुजाओों के बीच का
अोंतर ___ सेमी है । निम्ननिखित में से कौि सा नवकल्प नदए गए ररि स्थाि कर उसी क्रम में
भरिा सोंभव है ?
A. 7, 57, 11
B. 8, 64, 10
C. 9, 72, 13
D. 10, 80, 15
E. 12, 90, 18

Q4. Two boats X and Y cover 264 km and 432 km downstream distance in ___
hours and ___ hours respectively. Upstream speed of boat X is 3 km/hr less than
the upstream speed of boat Y whose speed in still water is 26 km/hr. Speed of
boat X in still water is 130% more than that of stream speed. The respective ratio
of downstream speed of boat X to downstream speed of boat Y is ___
respectively. Stream is flowing with speed ___ km/hr.
Which of the following option is possible to fill the given blanks in same order?
दर िावें X और Y िारा के अिुकूि 264 नकमी और 432 नकमी की दू री क्रमशः ___ घोंटे
और ___ घोंटे में तय करती हैं । िाव शाों त िािी में िाव X की गनत िारा की गनत से 130%
अनिक है । िाव X की िारा के अिुकूि गनत और िाव Y की िारा के अिुकूि गनत का
सोंबोंनित अिुिात क्रमशः ___ है । िारा ___ नकमी/घोंटा की गनत से बह रही है । निम्ननिखित में
से कौि सा नवकल्प नदए गए ररि स्थाि कर उसी क्रम में भरिा सोंभव है ?
A. 6, 12, 14:15, 20
B. 8, 12, 11:12,10
C. 6, 20, 12:13, 5
D. 8, 12, 10:11, 10
E. None of these

Q5. Anjali and Sapna started a business by investing Rs. 10500 and Rs. 7500
respectively. After 'x' months, Anjali added Rs. 3000 to her investment while
Sapna withdrew 1/5th of her investment. After next 'y' months, Anjali withdrew
Rs. 4500 while Sapna added an amount equal to 60% of her initial investment. At
the end of two - years partnership, profit earned by Anjali and Sapna is in the
ratio of 5 : 4 respectively. Which of the following statement is true?

अोंजनि और सििा िे रुिये का निवेश करके एक व्यवसाय शुरू नकया। 10500 और रु.
क्रमशः 7500. 'x' महीिे के बाद, अोंजनि िे रुिये जरडे । अििे निवेश से 3000 रु. निकाि
निए जबनक सििा िे अििे निवेश का 1/5 नहस्सा निकाि निया। अगिे 'y' महीिरों के बाद,
अोंजनि िे रुिये निकाि निए। 4500 जबनक सििा िे अििे शुरुआती निवेश के 60% के
बराबर रानश जरडी। दर साि की साझेदारी के अोंत में , अोंजनि और सििा िारा अनजषत िाभ
क्रमशः 5: 4 के अिुिात में है । निम्ननिखित में से कौि सा कथि सत्य है ?

A. 5x+4y =45
B. 14x+27y = 264
C. 12x+13y =220
D. 17x+19y = 185
E. None of these
Answer Key:

1. 89%
2. (A) - (I), (IV) and B - (II), (III)
3. 9, 72, 13
4. 8, 12, 11 : 12, 10
5. 14x + 27y = 264

Solution:

Q1.

Students admitted in college A & B in 2021 = x × 1.1 + 2x/1.5 = 1.1x + 1.6x = 2.7x
Students admitted in college C & D in 2021 = x × 1.3 + 2x/1.15 = 1.3x + 40x/23 =
699x/230
Required % = 2.7x/(699x/230) × 100 = 88.84% = 89%

Q2.

(A)
8X × (1.12 - 1) = 14Y
4X × (1.21 - 1) = 7Y
4X × 0.21 = 7Y
X : Y = 1 : 0.12 = 25 : 3

(B)
(400 - X)/Y = X/60
24000 - 60X = XY
X = 24000/(Y + 60)

From I)
a2 - 56a + 300 = 0
(a - 6)(a - 50) = 0
a = 6, 50
X = 50, Y = 6
X : Y = 50 : 6 = 25 : 3 (Satisfy A)
Y = 6, X = 24000/(6 + 60) = 24000/66 = 4000/11 ≠ 50 (Not Satisfy B)
From II) X = 240, Y = 40
X : Y = 240 : 40 = 6 : 1 (Not Satisfy A)
Y = 400, X = 24000/(40 + 60) = 24000/100 = 240 (Satisfy B)

From III)
3X + 4Y = 880
9X - 8Y = 240
2 × (3X + 4Y) + (9X - 8Y) = 2 × 880 + 240
6X + 8Y + 9X - 8Y = 1760 + 240
15X = 2000
X = 400/3
Y = (9 × 400/3 - 240)/8 = (1200 - 240)/8 = 960/8 = 120
X : Y = 400/3 : 120 = 10 : 9 (Not Satisfy A)
Y = 120, X = 24000/(120 + 60) = 24000/180 = 400/3 (Satisfy B)
From IV) X = 125, Y = 15
X : Y = 125 : 15 = 25 : 3 (Satisfy A)
Y = 15, X = 24000/(15 + 60) = 24000/75 = 320 (Not Satisfy B)
(A) - (I), (IV) and B - (II), (III)

Q3.

Side of Square = √576 = 24 cm


Breadth of Rectangle P = 24 cm
Length of Rectangle P = 24 × 9/8 = 27 cm
Area of Rectangle P = 24 × 27 = 648 cm2
Area of Square is less than Area of Rectangle P by = 648 - 576 = 72
Breadth of Rectangle Q = 24 × 7/8 = 21 cm
Length of Rectangle Q = (4 × 24 + 14)/2 - 21 = (96 + 14)/2 - 21 = 110/2 - 21 = 55 -
21 = 34 cm
Difference between sides of Rectangle Q = 34 - 21 = 13 cm
Area of Rectangle Q = 21 × 34 = 714 cm2
Area of Rectangle P is less than Area of Rectangle Q by = (714 - 648)/714 × 100 =
(9 + 29/119)%
9, 72, 13

Q4.

Speed of Boat Y in still water = 26 km/h


Speed of Boat X in still water = 26 - 3 = 23 km/hr
Speed of Stream = 23/2.3 = 10 km/hr
Time taken by Boat X to cover 264 km in downstream = 264/(23 + 10) = 264/33 =
8 hours
Time taken by Boat Y to cover 432 km in downstream = 432/(26 + 10) = 432/36 =
12 hours
Downstream Speed of Boat X : Downstream Speed of Boat Y
= (23 + 10) : (26 + 10) = 33 : 36 = 11 : 12
8, 12, 11 : 12, 10

Q5.

Investment of Anjali = 10500 × 24 + 3000 × (24 - x) - 4500 × (24 - x - y)


= (216000 + 1500x + 4500y) Rs.= 1500(144 + x + 3y) Rs.
Investment of Sapna = 7500 × 24 - 7500 × 1/5 × (24 - x) + 7500 × 0.6 × (24 - x - y)
= 7500 × (24 - 24/5 + x/5 + 14.4 - 0.6x - 0.6y) = 1500(168 - 2x - 3y) Rs.
1500(144 + x + 3y)/1500(168 - 2x - 3y) = 5/4
4 × (144 + x + 3y) = 5 × (168 - 2x - 3y)
576 + 4x + 12y = 840 - 10x - 15y
14x + 27y = 264
227

New Pattern Q/s- Statement Based

Q1. The question consists of three statements “I and II", decide whether the data
provided in the Statements are sufficient to answer the question.

Number of male employees in companies A, B and C is 'x + 100', 'x - 100' and 'x'
respectively. Total number of employees in companies A is '3y' which is equal to
that of C which is 25% less than total number of employees in B. Find the number
of female employees in company C.
Statement I : Ratio between the number of female employees in companies A, B
and C is 1 : 5 : 2.
Statement II : Number of female employees in company A is 400 less than that of
B. Number of female employees in company B is 300 more than that of C.

प्रश्न में तीि कथि "I और II" शानमि हैं , यह तय करें नक कथि में प्रदाि नकया गया डे टा
प्रश्न का उत्तर दे िे के निए ियाष प्त है या िहीों।

कोंििी A, B और C में िुरुर् कमषचाररयरों की सोंख्या क्रमशः 'x + 100', 'x - 100' और 'x'
है । कोंििी A में कमषचाररयरों की कुि सोंख्या '3y' है जर C के बराबर है जर B में कमषचाररयरों
की कुि सोंख्या से 25% कम है । कोंििी C में मनहिा कमषचाररयरों की सोंख्या ज्ञात कीनजए।

कथि I: कोंििी A, B और C में मनहिा कमषचाररयरों की सोंख्या के बीच का अिुिात 1: 5: 2


है ।

कथि II: कोंििी A में मनहिा कमषचाररयरों की सोंख्या B की तुििा में 400 कम है । कोंििी B
में मनहिा कमषचाररयरों की सोंख्या C की तुििा में 300 अनिक है।
A. Only I
B. Only II
C. Either I or II are alone sufficient
D. Both together are sufficient
E. None of them are sufficient

Q2. The question consists of two statements "I and II", decide whether the data
provided in the statement are sufficient to answer the question.
Three people Amit, Vivek and Chandan invested in a business. After 4 months,
Amit withdraws his initial amount and invested a second amount forrest of the
time, after 6 months from starting, Vivek withdraws his initial amount and
invested a second amount for rest of the time, and after 10 months from starting,
Chandan withdraws his initial and invested a second amount for rest of the time.
Find the share of Amit in the profit after 1 year.
Statement I : Second amount invested by Vivek and Chandan together is Rs.
24000, which is 40% of the total profit from the business after 1 years.

Statement II : Initial investment of Chandan, Amit and Vivek are in the ratio 1 : 2 :
1. Ratio of the second investment of Vivek and Chandan are in the ratio of 5 : 1
and ratio of the second investment of Chandan and Amit are in the ratio of 1 : 2.
प्रश्न में दर कथि "I और II" शानमि हैं , यह तय करें नक कथि में नदया गया डे टा प्रश्न का
उत्तर दे िे के निए ियाषप्त है या िहीों।

तीि िरगरों अनमत, नववेक और चोंदि िे एक व्यवसाय में निवेश नकया। 4 महीिे के बाद,
अनमत िे अििी प्रारों नभक रानश निकाि िी और शेर् समय के निए दू सरी रानश का निवेश
नकया, शुरुआत के 6 महीिे बाद, नववेक िे अििी प्रारों नभक रानश निकाि िी और बाकी
समय के निए दू सरी रानश का निवेश नकया, और शुरुआत के 10 महीिे बाद, चोंदि िे
निकासी कर िी अििी प्रारों नभक और शेर् समय के निए दू सरी रानश का निवेश नकया। 1
वर्ष के बाद िाभ में अनमत का नहस्सा ज्ञात कीनजये।

कथि I: नववेक और चोंदि िारा सोंयुि रूि से निवेश की गई दू सरी रानश रु. 24000, जर 1
वर्ष के बाद व्यवसाय से कुि िाभ का 40% है ।
कथि II: चोंदि, अनमत और नववेक के प्रारों नभक निवेश का अिुिात 1: 2: 1 है । नववेक और
चोंदि के दू सरे निवेश का अिुिात 5: 1 है और चोंदि और अनमत के दू सरे निवेश का
अिुिात है 1:2 का अिुिात.

A. Only I
B. Only II
C. Either I or II are alone sufficient
D. Both together are sufficient
E. None of them are sufficient

Q3. The question consists of two statements "I and II", decide whether the data
provided in the statement are sufficient to answer the question.
What is the cost price of Chair and Table together?
Statement I : A shopkeeper marked Table and Chair at 25% and 20% respectively
above its cost price. After giving 20% discount on each item, he had a loss of Rs. 6.
Cost price of Table is 2 times cost price of Chair.

Statement II : Selling price of Chair is 4% less than its cost price and selling price of
Table is 100% greater than the cost price of Chair. A shopkeeper has no loss/no
profit on selling Table but has a loss of Rs. 6 on selling Chair.
प्रश्न में दर कथि "I और II" शानमि हैं , यह तय करें नक कथि में नदया गया डे टा प्रश्न का
उत्तर दे िे के निए ियाषप्त है या िहीों।
कुसी और मेज का कुि िागत मूल्य क्या है ?
कथि I: एक दु कािदार िे मेज और कुसी िर उिकी िागत मूल्य से क्रमशः 25% और
20% अनिक मूल्य अोंनकत नकया। प्रत्येक वस्तु िर 20% छूट दे िे के बाद उसे रुिये की
हानि हुई। 6. टे बि का िागत मूल्य कुसी के िागत मूल्य का 2 गुिा है ।

कथि II: कुसी का नवक्रय मूल्य उसके िागत मूल्य से 4% कम है और टे बि का नवक्रय


मूल्य कुसी के िागत मूल्य से 100% अनिक है । एक दु कािदार कर टे बि बेचिे िर करई
हानि/करई िाभ िहीों हरता है िेनकि उसे रुिये की हानि हरती है । कुसी बेचिे िर 6 रु.
A. Only I
B. Only II
C. Either I or II are alone sufficient
D. Both together are sufficient
E. None of them are sufficient
Q4. The question consists of two statements "I and II", decide whether the data
provided in the statement are sufficient to answer the question.
In a group, there are 4 Probationary Officers, 6 Managers, some Clerks and some
Chief financial officers. A committee of six members is to be formed such that the
committee contains 1 Manger, 2 Clerks,1 Probationary Officer and 2 Chief
financial officers. Find the total number of ways in which the committee can be
formed.
Statement I : A committee of 4 members such that the committee contains 1
Clerk, 2 managers and 1 Probationary Officer can be formed in 120 different
number of ways.

Statement II : Probability of selecting one Chief financial officer from the group is
0.33.
प्रश्न में दर कथि "I और II" शानमि हैं , यह तय करें नक कथि में नदया गया डे टा प्रश्न का
उत्तर दे िे के निए ियाषप्त है या िहीों।
एक समूह में, 4 िररवीक्षािीि अनिकारी, 6 प्रबोंिक, कुछ क्लकष और कुछ मुख्य नवत्तीय
अनिकारी हैं । छह सदस्रों की एक सनमनत इस प्रकार बिाई जािी है नक सनमनत में 1
प्रबोंिक, 2 क्लकष, 1 िररवीक्षािीि अनिकारी और 2 मुख्य नवत्तीय अनिकारी हरों। सनमनत का
गठि नकतिे तरीकरों से नकया जा सकता है , इसकी कुि सोंख्या ज्ञात कीनजए।
कथि I: 4 सदस्रों की एक सनमनत, नजसमें 1 क्लकष, 2 प्रबोंिक और 1 िररवीक्षािीि
अनिकारी शानमि हैं , कर 120 अिग-अिग तरीकरों से बिाया जा सकता है ।

कथि II: समूह से एक मुख्य नवत्तीय अनिकारी के चयि की सोंभाविा 0.33 है ।


A. Only I
B. Only II
C. Either I or II are alone sufficient
D. Both together are sufficient
E. None of them are sufficient

Q5. The question consists of two statements "I and II", decide whether the data
provided in the statement are sufficient to answer the question.
Find the time taken by pipe 'R' alone to fill tank 'A'.
Statement I : Pipes 'R' and 'S' together can fill tank 'B' in 1 hour. The amount of
water emptied by pipe 'S' is equal to 3/5th of the amount of water filled by pipe
'Q', in the same time. Tank 'A' filled to 4/5th of its capacity will hold the same
water as tank 'B' filled to 70% of its capacity.
Statement II : Pipe 'P' alone takes 5 minutes more than pipe 'R' alone to fill tank
'A'. Pipe 'P' is 20% more efficient than pipe 'Q'.
प्रश्न में दर कथि "I और II" शानमि हैं , यह तय करें नक कथि में नदया गया डे टा प्रश्न का
उत्तर दे िे के निए ियाषप्त है या िहीों।
टैं क 'ए' कर भरिे में अकेिे िाइि 'आर' िारा निया गया समय ज्ञात कीनजए।

कथि I: िाइि 'आर' और 'एस' नमिकर टैं क 'बी' कर 1 घोंटे में भर सकते हैं । िाइि 'S' िारा
िािी नकए गए िािी की मात्ा, उसी समय में िाइि 'Q' िारा भरे गए िािी की मात्ा के
3/5वें नहस्से के बराबर है । टैं क 'ए' अििी क्षमता के 4/5 भाग तक भरा हुआ है , इसमें
उतिा ही िािी रहे गा नजतिा टैं क 'बी' में अििी क्षमता का 70% भरा हुआ है ।

कथि II: टैं क 'ए' कर भरिे में िाइि 'िी' अकेिे िाइि 'आर' से 5 नमिट अनिक िेता है ।
िाइि 'P', िाइि 'Q' से 20% अनिक कुशि है ।
A. Only I
B. Only II
C. Either I or II are alone sufficient
D. Both together are sufficient
E. None of them are sufficient
Answer Key:

1. A
2. D
3. C
4. D
5. D

Solution:

Q1.

In Company A, Total Employees = 3y


Male employee = (x + 100), Female employee = 3y - (x + 100) = (3y - x - 100)
In Company B, Total Employees = 3y/0.75 = 4y
Male employees = (x - 100), Female employees = 4y - (x - 100) = (4y - x + 100)
In Company C, Total Employees = 3y
Male employees = x, Female employees = 3y - x
From Statement I,
(3y - x - 100)/(4y - x + 100) = 1/5
15y - 5x - 500 = 4y - x + 100
11y - 4x = 600
(4y - x + 100)/(3y - x) = 5/2
8y - 2x + 200 = 15y - 5x
7y - 3x = 200
x = 400, y = 200
Female employees in company C = 3 × 200 - 400 = 600 - 400 = 200 (Statement I
alone is sufficient)

Q2.

From Statement I and Statement II


Total Profit = 24000/0.4 = 60000 Rs.
Second Amount invested by Vivek = 24000 × 5/6 = 20000 Rs.
Second Amount invested by Chandan = 24000 × 1/6 = 4000 Rs.
Second Amount invested by Amit = 4000 × 2 = 8000 Rs.
Investment of Amit = 2x × 4 + 8000 × 8 = (8x + 64000) Rs.
Investment of Chandan = x × 10 + 4000 × 2 = (10x + 8000) Rs.
Investment of Vivek = x × 6 + 20000 × 6 = (6x + 120000) Rs.
(24x + 192000) × 0.4 = 10x + 8000
24x + 192000 = 25x + 20000
x = 172000 Rs.
Investment of Amit = 8 × 172000 + 64000 = 1376000 + 64000 = 1440000 Rs.
Investment of Chandan = 10 × 172000 + 8000 = 1720000 + 8000 = 1728000 Rs.
Investment of Vivek = 6 × 172000 + 120000 = 1032000 + 120000 = 1152000 Rs.
Profit Ratio, Amit : Chandan : Vivek = 1440000 : 1728000 : 1152000 = 5 : 6 : 4
Profit Share of Amit = 60000 × 5/15 = 20000 Rs.
Both the statements together are sufficient

Q3.

From Statement I,
CP of Chair = x Rs.
CP of Table = 2x Rs.
(2x × 1.25 + x × 1.2) × 0.8 = (2x + x) - 6
(2.5x + 1.2x) × 0.8 = 3x - 6
3.7x × 0.8 = 3x - 6
3x - 2.96x = 6
0.04x = 6
x = 150
CP of Chair & Table both = x + 2x = 3 × 150 = 450 Rs. (Statement I alone is
sufficient)
From Statement II,
CP of Chair = 6/0.04 = 150 Rs.
CP of Table = 150 × 2 = 300 Rs.
CP of Chair & Table both = 150 + 300 = 450 Rs. (Statement II alone is sufficient)

Either is sufficient.

Q4.

Let Clerk = x, Chief Financial officers = y


From Statement I and Statement II together
x
C1 × 6C2 × 4C1 = 120
x × 15 × 4 = 120
x=2
y/(4 + 6 + 2 + y) = 1/3
3y = 12 + y
2y = 12
y=6
Number of Ways of forming a committee = 6C1 × 2C2 × 4C1 × 6C2 = 6 × 1 × 4 × 15 =
360
Both the Statements together are sufficient.

Q5.

From Statement I & Statement II together,


Efficiency of pipe Q = 5
Efficiency of pipe S = 3
Efficiency of pipe P = 5 × 1.2 = 6
Capacity of Tank B = x
Capacity of Tank A = 0.7x × 5/4 = 0.875x
Efficiency of R = x/60 + 3
0.875x/6 = 0.875x/(x/60 + 3) + 5
0.875x/6 × (x/60 + 3) = 0.875x + 5(x/60 + 3)
7x2/2880 + 7x/16 = 0.875x + x/12 + 15
7x2/2880 - 25x/48 - 15 = 0
7x2 - 1500x - 43200 = 0
(7x + 180)(x - 240) = 0
x = 240
Efficiency of R = x/60 + 3 = 240/60 + 3 = 4 + 3 = 7
Capacity of Tank A = 0.875 × 240 = 210
Time taken by R to fill the tank = 210/7 = 30 minute
Both the statements together are sufficient
228

Arithmetic Based Questions

Q1. Two typists of varying skills can do a work in 6 minutes if they work together.
If the 1st typist typed alone for 4 minutes and then the 2nd typist typed alone for 6
minutes, they would be left with 1/5th of the total work. How many minutes
would it take to complete the slower typist to complete the typing job working
alone? (In minutes)

अिग-अिग कौशि वािे दर टाइनिस्ट एक साथ काम करिे िर 6 नमिट में एक काम कर
सकते हैं । यनद िहिा टाइनिस्ट 4 नमिट के निए अकेिे टाइि करता है और नफर दू सरा
टाइनिस्ट 6 नमिट के निए अकेिे टाइि करता है , तर उिके िास कुि काम का 1/5 भाग
बचेगा। िीमे टाइनिस्ट कर अकेिे काम करते हुए टाइनिोंग का काम िूरा करिे में नकतिे
नमिट िगेंगे? (नमिटरों में)

A. 10
B. 12
C. 15
D. 20
E. None of these
Q2. A shopkeeper bought an article for Rs.4500 and than marked it at __% above
its cost price. Thereafter he allowed two discounts of __ % and 25% respectively
on its marked price and sold the article for Rs.300 less than its cost price.

एक दु कािदार िे 4500 रुिये में एक वस्तु िरीदी और उस िर िागत मूल्य से __% अनिक
अोंनकत नकया। इसके बाद उसिे अोंनकत मूल्य िर क्रमशः __% और 25% की दर छूटें दीों
और वस्तु कर उसके िागत मूल्य से 300 रुिये कम में बेच नदया।
I)25
II)10
III)5
IV)12
Which of the following can fill in the blanks?

निम्ननिखित में से कौि ररि स्थाि भर सकता है ?


A. Only I
B. Only II
C. Only III
D. Only IV
E. None of these

Q3. Quantity 1: A marks up the laptop by 20% profit and sells it at a discount of
15%. A’s net gain % is?
Quantity 2: if the cost price of 15 articles is equal to the selling price of 12
articles, find the gain percent.

मात्ा 1: A िैिटॉि कर 20% िाभ िर अोंनकत करता है और इसे 15% की छूट िर बेचता
है । A का शुि िाभ % है ?

मात्ा 2: यनद 15 वस्तुओों का िागत मूल्य 12 वस्तुओों के नवक्रय मूल्य के बराबर है , तर िाभ
प्रनतशत ज्ञात करें ।

A. Q1>Q2
B. Q1<Q2
C. Q1<=Q2
D. Q2<=Q2
E. Q1=Q2 or no relation can be established
Q4. A contractor employed ‘a’ number of people in order to finish the work in 4
days. But due to some reason, everyday exactly 5 people didn’t come to work. As
a result the whole work was completed in 3/4th more time than usual. Find the
value of ‘a’.

एक ठे केदार िे 4 नदिरों में काम िूरा करिे के निए 'ए' िरगरों कर नियरनजत नकया। िेनकि
नकसी कारण से हर ररज ठीक 5 िरग काम िर िहीों आते थे. िररणामस्वरूि िूरा कायष
सामान्य से 3/4 अनिक समय में िूरा हर गया। 'ए' का माि ज्ञात कीनजए।
A. 50
B. 40
C. 35
D. 25
E. 20
Q5-6. A family has 11 members (A,B,C,D,E,F,G,H,I,J and K). The age of one of the
members A is 4 years younger than the age of B, who is 28 years old. When the
ages of these two members is excluded, the average age of the remaining
members of the family becomes 2 years less than the average age when all the
members are included.

एक िररवार में 11 सदस् हैं (ए, बी, सी, डी, ई, एफ, जी, एच, आई, जे और के)। सदस्रों में
से एक A की आयु B की आयु से 4 वर्ष कम है , जर 28 वर्ष का है । जब इि दर सदस्रों की
आयु हटा दी जाती है , तर िररवार के शेर् सदस्रों की औसत आयु सभी सदस्रों कर शानमि
करिे िर औसत आयु से 2 वर्ष कम हर जाती है ।
Q5. If it was given that the age of A is 4 years more than the age of B, and the
average age of the remaining members of the family becomes 2 years less than
the average age when all the members are included. What is the difference
between the original average and the new average?

यनद यह नदया जाए नक A की आयु B की आयु से 4 वर्ष अनिक है , और सभी सदस्रों कर


शानमि करिे िर िररवार के शेर् सदस्रों की औसत आयु औसत आयु से 2 वर्ष कम हर
जाती है । मूि औसत और िए औसत के बीच क्या अोंतर है ?
A. 1 year
B. 3 years
C. 5 years
D. Cannot be determined
E. None of these

Q6. The average C, D, E is 12 years and the average age of F, H, I, J and K is 14


years. Find the age of G.

C, D, E की औसत आयु 12 वर्ष है और F, H, I, J और K की औसत आयु 14 वर्ष है। G की


आयु ज्ञात कीनजये.
A. 15 years
B. 13 years
C. 17 years
D. 18 years
E. Cannot be determined
Answer Key:
1. A
2. E
3. B
4. C
5. E
6. C

Solution:
Q1. Let the first typist take x minutes and second typist take y minutes in doing
the work.
Then,
1/x +1/y = 1/6 --- (I)
Again,
4/x + 6/y =4/5 --- (II)
Now checking through option
Option I
When x = 10, then
1/10 +1/y = 1/6
1/y = 1/15
Y =15
Putting in II
LHS = 4/106/15 = 4/5 =RHS

It is given that CP is 4500 and SP = 4500-300 = 4200


So CP: SP = 45:42 = 15:14
Now to fill in the blanks (which have the same amount as only 1 option is given)
check from options and the one which has the same CP:SP ratio is the answer.
From I
CP/SP = (10/11)*(10/9)*(4/3) = 400/297 (not = 15:14)
So option I doesn’t fit.
Similarly, we get,
from II = 64:45
from III = 1600:1197
from IV= 625:462
Thus None of these will fill in the blanks.

Q3.
Quantity 1
Net gain % = {20-15-[(20*15)/100}} = 20-18 = 2%
Quantity 2
Let CP of 1 article be Rs.1 then CP of 15 articles = Rs.15 and of 12 articles is Rs.12
SP of 12 = CP of 15 =Rs. 15
Gain % = [(15-112)/12]*100 =( ¼)*100 = 25%
Q1<Q2

Q4. From options


Option C is correct, i.e
35*4 = 35+30+25+20+15+10+5
140=140 (LHS = RHS)
Q 5-6. Age of B = 28 years, A = 28-4 = 24 years
Let the age of all 9 members excluding A and B be ‘a’, so sum of their ages will be
‘9a’
 (9a + 28+24 /11) – a = 2
 9a + 52 – 11a = 22
 52-2a = 22
 2a = 30
 a = 15 (Average age of 9 members excluding A and B)
Also, 9a = 15*9 = 135
Therefore average age of all 11 members is
= (135+28+24)/ 11 = 187/11 = 17

Q5. As per the condition in this questions A’s new age will be = 28+4 = 32
So Again,
=(9a + 28+32 /11) – a = 2
=9a + 60 – 11a = 22
=60-2a = 22
=2a = 38
=a = 19 (Average age of 9 members excluding A and B)
Also, 9a = 19*9 = 171
Therefore average age of all 11 members is
= (171+28+32)/ 11 = 231/11 = 21 (new average)
Difference = 21-17 = 4 years
Q6. As we calculated above, the sum of ages of the other 9 members was 135
years.
The average C, D, E is 12 years, so sum is = 12*3 = 48 years
And, the average age of F, H, I, J and K is 14 years, so sum is = 14*5 = 70 years
Thus the age of G = 135 – 70 – 48 = 17 years
229

Data Interpretation

Directions: Answer the following questions based on the information provided


below.
The graph given below represents the number of people who applied for various
positions i.e. Clerk, PO, SO and Scale I officer, in five different banks in 2022. Pie
chart represents the number of people who qualified and the bar graph shows
the ratio of number of people who applied for the position of Clerk to that of
number of people who applied for the position of PO.

निदे श: िीचे दी गई जािकारी के आिार िर निम्ननिखित प्रश्नरों के उत्तर दीनजए।

िीचे नदया गया ग्राफ़ 2022 में िाों च अिग-अिग बैंकरों में नवनभन्न िदरों यािी क्लकष, िीओ,
एसओ और स्केि I अनिकारी के निए आवेदि करिे वािे िरगरों की सोंख्या कर दशाष ता है ।
िाई चाटष यरग्य िरगरों की सोंख्या कर दशाष ता है और बार ग्राफ़ सोंख्या के अिुिात कर दशाष ता
है । क्लकष के िद के निए आवेदि करिे वािे िरगरों की सोंख्या से िेकर िीओ के िद के निए
आवेदि करिे वािे िरगरों की सोंख्या तक।

Candidates Applied in Banks (in


hundreds)
14
12 12.5 13
10
8 8.4 9
6
7
4
2
0
Axis BOB Canara Deutsche PNB
% Qualified

54
75
Canara
PNB
BOB
66
Axis
72 Deutsche

60

Ratio (out of those who Qualified)

PNB

Deutsche

Canara Clerk
PO
BOB

Axis

0 2 4 6 8 10
Q1. The number of PO’s selected in Canara Bank increased by 20% in 2022 as
compared to 2021 but the ratio of Selected Clerk to PO remained same in both
the years. If the number of people who got selected as PO in 2022 is 60, then
what is the difference between the number of Clerks in 2021 and 2022?

केिरा बैंक में चयनित िीओ की सोंख्या 2021 की तुििा में 2022 में 20% बढ़ गई िेनकि
चयनित क्लकष और िीओ का अिुिात दरिरों वर्ों में समाि रहा। यनद 2022 में िीओ के
रूि में चयनित हरिे वािे िरगरों की सोंख्या 60 है , तर 2021 और 2022 में क्लकों की सोंख्या
के बीच क्या अोंतर है ?

A. 25
B. 30
C. 20
D. Cannot be determined
E. None of these
Q2. The number of people got selected as SO is half the number of people
selected as Clerk in Axis Bank. If the number of people who applied for Scale I
officer in Axis Bank is 400 and out of this 62.5% got selected as Scale I officer,
then find the number of people who got selected as Clerks in Axis Bank.

एसओ के रूि में चुिे गए िरगरों की सोंख्या एखिस बैंक में क्लकष के रूि में चुिे गए िरगरों
की सोंख्या की आिी है । यनद एखिस बैंक में स्केि I अनिकारी के निए आवेदि करिे वािे
िरगरों की सोंख्या 400 है और इसमें से 62.5% स्केि I अनिकारी के रूि में चुिे गए हैं , तर
एखिस बैंक में क्लकष के रूि में चुिे गए िरगरों की सोंख्या ज्ञात कीनजए।

A. 330
B. 320
C. 310
D. 300
E. None of these
Q3. If the ratio of number of people selected as Clerk in BOB and Deutsche is 3:1,
respectively, and the sum of people selected as Clerk and PO in BOB was 560,
then find the number of people selected as PO in Deutsche.

यनद BOB और डॉयचे में क्लकष के रूि में चुिे गए िरगरों की सोंख्या का अिुिात क्रमशः 3:1
है , और BOB में क्लकष और PO के रूि में चुिे गए िरगरों का यरग 560 था, तर डॉयचे में PO
के रूि में चुिे गए िरगरों की सोंख्या ज्ञात कीनजए।
A. 70
B. 85
C. 210
D. 140
E. Cannot be determined

Q4. The difference between the number of people selected as Clerk and PO in
PNB is 30 and the number of people selected as Scale I officer in PNB is 100. If the
number of people applied as SO is 378, then find the number of people who
didn’t qualify as SO in PNB.

िीएिबी में क्लकष और िीओ के रूि में चुिे गए िरगरों की सोंख्या 30 है और िीएिबी में
स्केि I अनिकारी के रूि में चुिे गए िरगरों की सोंख्या 100 है । यनद एसओ के रूि में
आवेदि करिे वािे िरगरों की सोंख्या 378 है , तर आवेदि करिे वािे िरगरों की सोंख्या ज्ञात
करें मैं िीएिबी में एसओ के रूि में यरग्य िहीों हों ।

A. 144
B. 158
C. 129
D. 164
E. None of these
Q5. If in 2023, in Axis Bank, 10% of clerks selected in 2022 got promoted to PO
and 25% of the PO’s selected in 2022 got promoted to Scale I officer, what is the
difference in number of Clerks and PO’s in 2023? (Use data from Q2)

यनद 2023 में, एखिस बैंक में, 2022 में चयनित 10% क्लकों कर PO में िदरन्नत नकया गया
और 2022 में चयनित PO में से 25% कर स्केि I अनिकारी के रूि में िदरन्नत नकया गया,
तर 2023 में क्लकष और PO की सोंख्या में क्या अोंतर है ? (Q2 से डे टा का उियरग करें )

A. 120
B. 90
C. 100
D. 60
E. Cannot be determined
Answer Key:

1. C
2. D
3. C
4. E
5. B

Solution:
For Axis Bank
Applied = 1250
Qualified (Selected) = 1250*72% = 900
Similarly,
Applied Selected
Axis Bank 1250 900
BOB 1300 780
Canara Bank 700 378
Deutsche Bank 840 630
PNB 900 594
Total 4990 3282

Q1. In Canara Bank


Selected PO in 2021 = 60*100/120 = 60*5/6 = 50
Selected Clerk in 2022 = 60*2/1 = 120
Selected Clerk in 2021 = 50*2/1 = 100
Thus, Difference = 120-100 = 20
Q2. In Axis Bank
Selected Scale I Officer = 400*62.5% = 250
Let the Number of clerks selected be 3u, PO be 2u, then SO = 3u (1/2) = 1.5u
Clerk + PO + SO = 900-250 = 650
Or, 3u + 2u + 1.5u = 6.5u = 650
So, 1u = 100
Thus the number of clerks who got selected = 3*100 = 300
Q3. Let the number of clerks selected in BOB be 3u & PO be 5u
Thus number of selected Clerks in Deutsche Bank = 3u*(1/3)= 1u
Now, it is given that,
3u + 5u = 560
8u = 560
1u = 70 (Number of selected Clerks)
Then, Number of people selected as PO in Deutsche bank = 70*3/1 = 210
Q4. In PNB
Let number of selected Clerks be 5u and POs be 4u
Then, 5u-4u =30
1u = 30
Selected Clerks = 5*30 = 150
Selected POs = 4*30 = 120
Selected SOs = 594-[150+120+100] = 594 – 370 = 224
People not Selected as SO = 378-224 = 154
Q5. In Axis bank
Number of clerks in 2023 = 300*90% = 270
Number of PO in 2023 = [200*75%] + [300*10%] = 150+30 = 180
Difference = 270-180 = 90
230

Data Interpretation

Directions: Answer the following questions based on the information provided


below.
A seller bought certain number of Math Books, English Books and Science Books
from a publishing house. Number of Math Books is 220% more than number of
English Books. Number of Math Books is 24 less than the number of Science
Books. Cost price of each science Book is 220 less than the cost price of each
Math Book. Each Science Book, English Book and Math Book marked at a few
percent above their respective cost prices. The bookseller sold all Science Books
together at Rs 80000. A discount is given on the marked price of each Science
Book. Discount (in rupees) given on each English Book and profit (in rupees)
earned on it are equal. Selling price of each Math Book and each Science Book are
in the ratio 7: 4 respectively. Selling price of each English Book is Rs 1500 which is
Rs 800 more than the selling price of each Math Book. Cost price of all English
Books together is Rs 66000. Average of marked price of each Math Book and each
English Book taken together is Rs 1305.

निदे श: िीचे दी गई जािकारी के आिार िर निम्ननिखित प्रश्नरों के उत्तर दीनजए।

एक नवक्रेता िे एक प्रकाशि गृह से निनित सोंख्या में गनणत की नकताबें , अोंग्रेजी की नकताबें
और नवज्ञाि की नकताबें िरीदीों। गनणत की िुस्तकरों की सोंख्या अोंग्रेजी िुस्तकरों की सोंख्या से
220% अनिक है। गनणत की िुस्तकरों की सोंख्या नवज्ञाि की िुस्तकरों की सोंख्या से 24 कम
है । प्रत्येक नवज्ञाि की िुस्तक का िागत मूल्य प्रत्येक गनणत की िुस्तक के िागत मूल्य से
220 कम है । प्रत्येक नवज्ञाि की नकताब, अोंग्रेजी की नकताब और गनणत की नकताब िर
उिके सोंबोंनित िागत मूल्य से कुछ प्रनतशत अनिक मूल्य अोंनकत है । िुस्तक नवक्रेता िे सभी
नवज्ञाि िुस्तकें एक साथ 80000 रुिये में बेचीों। प्रत्येक नवज्ञाि िुस्तक के अोंनकत मूल्य िर
छूट दी गई है । प्रत्येक अोंग्रेजी िुस्तक िर दी गई छूट (रुिये में) और उस िर अनजषत िाभ
(रुिये में) बराबर है । प्रत्येक गनणत की िुस्तक और प्रत्येक नवज्ञाि की िुस्तक का नवक्रय
मूल्य क्रमशः 7: 4 के अिुिात में है । प्रत्येक अोंग्रेजी िुस्तक का नवक्रय मूल्य 1500 रुिये है
जर गनणत की प्रत्येक िुस्तक के नवक्रय मूल्य से 800 रुिये अनिक है । सभी अोंग्रेजी नकताबरों
का िागत मूल्य एक साथ 66000 रुिये है। प्रत्येक गनणत की नकताब और प्रत्येक अोंग्रेजी
नकताब का एक साथ नमिाकर अोंनकत मूल्य का औसत 1305 रुिये है ।

Q1. What is the markup percentage of all the 3 books?

तीिरों िुस्तकरों का माकषअि प्रनतशत क्या है ?

A. 20%
B. 25%
C. 40%
D. Cannot be determined
E. None of these
Q2. What is the average of discount given on each of all three books?

तीिरों िुस्तकरों में से प्रत्येक िर दी गई छूट का औसत क्या है ?

A. Rs.163.33
B. Rs.133.33
C. Rs.113.33
D. Rs.116.66
E. None of these
Q3. If marked price of 20% of English Books is equal to the cost price of each Math
Book, marked price of 20 English Books is equal to the Cost price of each English
Book and marked price of remaining English Books is equal to the marked price of
each Science Book, then what is the difference between new marked price of all
English Books together and their actual marked price?

यनद 20% अोंग्रेजी िुस्तकरों का अोंनकत मूल्य प्रत्येक गनणत की िुस्तक के िागत मूल्य के
बराबर है , तर 20 अोंग्रेजी िुस्तकरों का अोंनकत मूल्य प्रत्येक अोंग्रेजी िुस्तक के िागत मूल्य के
बराबर है और शेर् अोंग्रेजी िुस्तकरों का अोंनकत मूल्य प्रत्येक गनणत की िुस्तक के अोंनकत
मूल्य के बराबर है। प्रत्येक नवज्ञाि िुस्तक, तर सभी अोंग्रेजी िुस्तकरों के एक साथ िए अोंनकत
मूल्य और उिके वास्तनवक अोंनकत मूल्य के बीच क्या अोंतर है ?
A. 52930
B. 55820
C. 57540
D. 58150
E. Cannot be determined
Q4. If the seller also bought Computer books, in which cost price of a book is 25%
more than Cost price of each Science book and each Computer book is sold at
marked price which is 12.5% more than its cost price, what is the number of
Computer books bought by the seller? It is given that he generated total revenue
of Rs.54000 on selling computer books and he sells all the books he bought.

यनद नवक्रेता िे कोंप्यूटर की नकताबें भी िरीदीों, नजसमें एक नकताब का िागत मूल्य प्रत्येक
नवज्ञाि की नकताब के िागत मूल्य से 25% अनिक है और प्रत्येक कोंप्यूटर की नकताब
अोंनकत मूल्य िर बेची जाती है , जर उसके िागत मूल्य से 12.5% अनिक है , तर िुस्तकरों की
सोंख्या क्या है ? नवक्रेता िारा िरीदी गई कोंप्यूटर िुस्तकें? यह नदया गया है नक उसिे
कोंप्यूटर नकताबें बेचिे िर कुि 54000 रुिये का राजस्व अनजषत नकया और उसिे िरीदी
गई सभी नकताबें बेच दीों।

A. 120
B. 100
C. 80
D. 40
E. Cannot be determined
Solution:
SP of 1 English book = 1500 Rs.
SP of 1 Math book = 1500-800 = 700 Rs.
SP of 1 Science book = 700*4/7 = 400 Rs.
Number of Science Book = Total Revenue Earned/SP of 1 book = 80000/400 = 200
Number of Math Book = 200 -24 = 176
Number of English Book = 176*100/320 = 55
CP of 1 English Book = Total Revenue/Total No. of Books = 66000/55 = 1200RS.
Now it is given that for English books,
Discount on 1 book = Profit on 1 book = 1500-1200 = 300Rs.
Thus MP of 1 English Book is (SP + Discount) = 1500+300 = 1800Rs.
MP of 1 Math Book = (1305*2) – 1800 = 810Rs.
Also, it is given MP % is same for all 3 books, i.e.
= {[1800-1200]/1200}*100 = 600/12 = 50%
Thus, CP of 1 Math Book = 810*2/3 = 540Rs.
&, CP of 1 Science Book = 540-220= 320Rs.
MP of 1 Science book = 320*3/2 = 480Rs.
Math Book English Book Science Book
Quantity (No. of 176 55 200
books)
Cost Price (1 book) 540 1200 320
Selling price(1 book) 700 1500 400
Marked Price(1 book) 810 1800 480

Q1. Markup % of all 3 is 50%.


Q2. Total Discount = (810-700)+ (1800-1500)+ (480-400) = 110+300+80 = 490Rs.
Average Discount = 490/3 = 163.33Rs

Q3. New Total MP = {[(55/5)*540 + [20*1200] + [(55-20-55/5]*480]


= [(11*540) + (24000) + (24*480)] =5940+24000+11520
= Rs. 41460
Original Total MP = 55*1800 = 99000 rs
Difference in MP (Old-New) = 99000-41460 =57540 Rs
Q4. CP of 1 Computer Book = 320*5/4 = 400rs.
MP of 1 Computer Book = 400*9/8 = 450Rs. (Here MP = SP)
Number of Computer Books = 54000/450 = Rs.120

You might also like